Compare 8/3 and 19/8.

Answers

Answer 1

Answer:

8/3 > 19/8

Step-by-step explanation:

Get a common denominator

8/3 *8/8 = 64/24

19/8 *3/3 =57/24

Comparing the numerators

64>57

8/3 > 19/8


Related Questions

Use the given information to determine which of the following relationships
can be proved and why.
L= 20
ME ZP
ML = PO
A. ALMN - A OPQ, because of AAS.
B. ALMNE A OPQ, because of ASA.
C. We cannot prove any relationship based on these data.
D. ALMN=A OPQ, because of SAS,

Answers

Answer:

B. ∆LMN ≅ ∆OPQ because of ASA

Step-by-step explanation:

Two triangles are congruent if two angles and an included side of one triangle are congruent to two corresponding angles and a corresponding included side of the other.

From the information given, we have:

Two angles (<L and <M) in ∆LMN that are congruent to two corresponding angles (<O and <P) in ∆OPQ.

Also, included side in both triangles are congruent (ML ≅ PO).

Therefore, ∆LMN ≅ ∆OPQ by the ASA Theorem.

Select the statement that best justifies the conclusion based on the given information.

If a(b + c) = d, then ab + ac = d.

associative
commutative
distributive
closure

Answers

Answer:

distributive

Step-by-step explanation:

a(b + c)=ab + ac

it's distributive one

Charity is planting trees along her driveway, and she has 6 pine trees and 6 willows to plant in one row. What is the probability that she randomly plants the trees so that all 6 pine trees are next to each other and all 6 willows are next to each other

Answers

Answer:

0.0022 = 0.22% probability that she randomly plants the trees so that all 6 pine trees are next to each other and all 6 willows are next to each other.

Step-by-step explanation:

A probability is the number of desired outcomes divided by the number of total outcomes.

In this question, the elements are arranged, so we have to use the arrangements formula.

Arrangements formula:

The number of possible arrangements of n elements is:

[tex]A_{n} = n![/tex]

Desired outcomes:

Pine trees(6!) then the willows(6!) or

Willows(6!) then the pine trees(6!). So

[tex]D = 2*6!*6! = 1036800 [/tex]

Total outcomes:

12 trees, so:

[tex]T = 12! = 479001600 [/tex]

What is the probability that she randomly plants the trees so that all 6 pine trees are next to each other and all 6 willows are next to each other?

[tex]p = \frac{D}{T} = \frac{1036800 }{479001600 } = 0.0022[/tex]

0.0022 = 0.22% probability that she randomly plants the trees so that all 6 pine trees are next to each other and all 6 willows are next to each other.

Find y' for the following.​

Answers

Answer:

[tex]\displaystyle y' = \frac{5x - 2xy^2}{2y(x^2 - 3y)}[/tex]

General Formulas and Concepts:

Calculus

Differentiation

DerivativesDerivative Notation

Derivative Property [Multiplied Constant]:                                                           [tex]\displaystyle \frac{d}{dx} [cf(x)] = c \cdot f'(x)[/tex]

Derivative Property [Addition/Subtraction]:                                                         [tex]\displaystyle \frac{d}{dx}[f(x) + g(x)] = \frac{d}{dx}[f(x)] + \frac{d}{dx}[g(x)][/tex]

Basic Power Rule:

f(x) = cxⁿf’(x) = c·nxⁿ⁻¹

Derivative Rule [Product Rule]:                                                                             [tex]\displaystyle \frac{d}{dx} [f(x)g(x)]=f'(x)g(x) + g'(x)f(x)[/tex]

Derivative Rule [Chain Rule]:                                                                                 [tex]\displaystyle \frac{d}{dx}[f(g(x))] =f'(g(x)) \cdot g'(x)[/tex]

Implicit Differentiation

Step-by-step explanation:

Step 1: Define

Identify

[tex]\displaystyle 5x^2 - 2x^2y^2 + 4y^3 - 7 = 0[/tex]

Step 2: Differentiate

Implicit Differentiation:                                                                                 [tex]\displaystyle \frac{dy}{dx}[5x^2 - 2x^2y^2 + 4y^3 - 7] = \frac{dy}{dx}[0][/tex]Rewrite [Derivative Property - Addition/Subtraction]:                                 [tex]\displaystyle \frac{dy}{dx}[5x^2] - \frac{dy}{dx}[2x^2y^2] + \frac{dy}{dx}[4y^3] - \frac{dy}{dx}[7] = \frac{dy}{dx}[0][/tex]Rewrite [Derivative Property - Multiplied Constant]:                                   [tex]\displaystyle 5\frac{dy}{dx}[x^2] - 2\frac{dy}{dx}[x^2y^2] + 4\frac{dy}{dx}[y^3] - \frac{dy}{dx}[7] = \frac{dy}{dx}[0][/tex]Basic Power Rule [Product Rule, Chain Rule]:                                             [tex]\displaystyle 10x - 2 \Big( \frac{d}{dx}[x^2]y^2 + x^2\frac{d}{dx}[y^2] \Big) + 12y^2y' - 0 = 0[/tex]Basic Power Rule [Chain Rule]:                                                                     [tex]\displaystyle 10x - 2 \Big( 2xy^2 + x^22yy' \Big) + 12y^2y' - 0 = 0[/tex]Simplify:                                                                                                         [tex]\displaystyle 10x - 4xy^2 - 4x^2yy' + 12y^2y' = 0[/tex]Isolate y' terms:                                                                                             [tex]\displaystyle -4x^2yy' + 12y^2y' = 4xy^2 - 10x[/tex]Factor:                                                                                                           [tex]\displaystyle y'(-4x^2y + 12y^2) = 4xy^2 - 10x[/tex]Isolate y':                                                                                                       [tex]\displaystyle y' = \frac{4xy^2 - 10x}{-4x^2y + 12y^2}[/tex]Simplify:                                                                                                         [tex]\displaystyle y' = \frac{5x - 2xy^2}{2y(x^2 - 3y)}[/tex]

Topic: AP Calculus AB/BC (Calculus I/I + II)

Unit: Differentiation

Book: College Calculus 10e

NO LINKS OR ANSWERING QUESTIONS YOU DON'T KNOW!!! Given this frequency chart of 1490 passengers from the Titanic who died, choose the class(es) whose relative frequency would comprise just under, 1/2 of a pie chart​

Answers

Answer:

b and e

Step-by-step explanation:

Second and Third which gives in total:

0.112 + 0.354 = 0.466

This is under 1/2 and greater than Crew.

Hellp PLZZzzzzzzzxxxmxxxxxxxxxx

Answers

Answer:

12. We use Rational numbers when the number is in P/q form

we don't use integers because they are not in p/q form

13.  Aron is wrong . he is not correct

an opposite of rational number can out be  negative it should be irrational number

opposite of integers are negative  that's why

*so opposite of rational numbers will ne irrational numbers not negative

14. Greatest to least

   -3.02 , -4 , -4.09 , -4.32, -4,35 , -5.11

Help me with this question please...


Each of the following statements is true or false. Which statements are true?

A. A triangle where at least two angles are acute is called an acute triangle.
B. Some polygons are neither convex nor concave.
C. The sum of the interior angles of a concave pentagon is $540^{\circ}.$
D. The interior angles of a regular $1000$-gon are greater than the interior angles of a regular $100$-gon.
E. The exterior angles of a regular $1000$-gon are greater than the exterior angles of a regular $100$-gon.

Answers

9514 1404 393

Answer:

  A. False

  B. False

  C. True

  D. True

  E. False

Step-by-step explanation:

A. False -- any triangle has at least two acute angles, whether it is acute, right, or obtuse.

B. False -- by definition, any polygon that is not convex is concave.

C. True -- the angle sum is the same regardless of whether the pentagon is convex or concave. (Provided it is a "simple" polygon, with no crossing sides.)

D. True -- the measure of the interior angle of a regular polygon increases as the number of sides increases. (see E)

E. False -- the exterior angles of a regular polygon are 360° divided by the number of sides. As the number of sides increases, the measure of each exterior angle decreases. (Interior angles are the supplement of exterior angles, so they increase as the number of sides increases.)

11
Select the correct answer.
Which expression is equivalent to the given expression?
In(2e/x)
O A. In 2 – In x
OB. 1 + In 2 - In x
Oc. In 2 + In x
OD. In 1 + In 2 - In
Reset
Next

Answers

Answer:

B. 1 + ln 2 - ln x

General Formulas and Concepts:

Algebra II

Natural logarithms ln and Euler's number eLogarithmic Property [Multiplying]:                                                                     [tex]\displaystyle log(ab) = log(a) + log(b)[/tex] Logarithmic Property [Dividing]:                                                                         [tex]\displaystyle log(\frac{a}{b}) = log(a) - log(b)[/tex]

Step-by-step explanation:

Step 1: Define

Identify

[tex]\displaystyle ln(\frac{2e}{x})[/tex]

Step 2: Simplify

Expand [Logarithmic Property - Dividing]:                                                      [tex]\displaystyle ln(\frac{2e}{x}) = ln(2e) - ln(x)[/tex]Expand [Logarithmic Property - Multiplying]:                                                  [tex]\displaystyle ln(\frac{2e}{x}) = ln(2) + ln(e) - ln(x)[/tex]Simplify:                                                                                                             [tex]\displaystyle ln(\frac{2e}{x}) = ln(2) + 1 - ln(x)[/tex]Rewrite:                                                                                                             [tex]\displaystyle ln(\frac{2e}{x}) = 1 + ln(2) - ln(x)[/tex]

A box contains two blue cards numbered 1 and 2, and three green numbered 1 through 3. A blue card ins picked, followed by a green card. Select sample space for such experiment
a) {1, 1), (1, 2, (1, 3)(2, 1), (2, 2), (2, 3)}
b) {(1, 1)(1, 2), (2, 1), (2, 2), (3, 1), (3, 2)}
c) {5}
d) {6}

Answers

Answer:

The answer is a.

The second term in a geometric sequence is 50. The forth term in the same sequence is 112.5. what is the common ratio in this sequence?

Answers

Answer:

1.5

Step-by-step explanation:

Let the first term be a and the common ratio be r

ATQ, ar=50 and ar^3=112.5, divide these two. r^2=2.25, r=1.5

What is the answer for 75% of test takers whovscored below average withou an unknown mean and standard deviation

Answers

Answer:

sir she hey Jen Jen Jenn receive surge

Answer:

Hello,

Step-by-step explanation:

z=0.7734

p(z<?)=0.75 ==> ?=0.7734

PLEASE HELPPPPPPPPPP

Answers

Answer: SORRY NEED AN ACCOUNT ON - 10

Step-by-step explanation:

To resolve the proposed issue, an explanation is needed in which the subject is addressed

can anybody help with this ?

Answers

Answer:(

fx).(gx)=D. -40x^3+25x^2+45

Step-by-step explanation:

18. The function f(x) = 4x - 8 is reflected across the y-axis, resulting in a new
function, g(x). Write the equation of g(x).

Please explain the steps!! ❤️

Answers

The equation of the reflected function across the y-axis is g(x) = -4x - 8.

What is a function?

A statement, principle, or policy that creates the link between two variables is known as a function. Functions are found all across mathematics and are required for the creation of complex relationships.

The function f(x) = 4x - 8 is reflected across the y-axis.

The function g(x) will be given by putting the negative x in place of x. Then the reflected function is obtained.

g(x) = -4x - 8

Then the equation of the reflected function across the y-axis is g(x) = -4x - 8.

The graph of the reflected graph is given below.

More about the function link is given below.

https://brainly.com/question/5245372

#SPJ2

A flower bed is in the shape of a triangle with one side twice the length of the shortest side and a third side is 22 more than the length of the shortest side. Find the dimensions if the perimeter is 182 feet.

Answers

Answer:40, 80 and 62

Step-by-step explanation:

182-22= 160

160/4 = 40 so,

Shortest side is 40

Longest is 80

Third side is 62

Part b c and d please help

Answers

Answer:

b) Y =5.73X +4.36

C)  =5.73225*(21)X +4.359

    124.73625

D) 163.728 = 5.73X +4.36  

     X = (163.728 - 4.36)/5.73

     X = 27.81291449

  Year would be 2027

Step-by-step explanation:

x1 y1  x2 y2

4 27.288  16 96.075

   

(Y2-Y1) (96.075)-(27.288)=   68.787  ΔY 68.787

(X2-X1) (16)-(4)=    12  ΔX 12

   

slope= 5 41/56    

B= 4 14/39    

   

Y =5.73X +4.36      

Inverse Function Question

Determine the expression of f^-1(x) for f(x)=e^x

Answers

First, find the inverse of f,

[tex]y=e^x[/tex]

[tex]x=e^y[/tex]

Now take the natural logarithm on both sides,

[tex]\ln x=\ln e^y\implies f^{-1}(x)=\boxed{\ln(x)}[/tex]

Second, find the inverse of g,

[tex]y=5x\implies g^{-1}(x)=\boxed{\frac{x}{5}}[/tex]

Now take their composition,

[tex](g\circ f)(x)=g(f(x))=\frac{\ln(x)}{5}[/tex]

Let [tex]y=\frac{\ln(x)}{5}[/tex], now again find the inverse,

[tex]x=\frac{\ln(y)}{5}[/tex]

[tex]5x=\ln y[/tex]

exponentiate both sides to base e,

[tex]e^{5x}=e^{\ln y}\implies (g\circ f)^{-1}(x)=\boxed{e^{5x}}[/tex]

Hope this helps :)

A chemical company makes two brands of antifreeze. The first brand is

55%

pure antifreeze, and the second brand is

80%

pure antifreeze. In order to obtain

130

gallons of a mixture that contains

70%

pure antifreeze, how many gallons of each brand of antifreeze must be used?

Answers

9514 1404 393

Answer:

52 gallons of 55%78 gallons of 80%

Step-by-step explanation:

Let x represent the quantity of 80% solution. Then the quantity of 55% solution is (130-x) and the total amount of antifreeze in the mix is ...

  0.55(130 -x) +0.80(x) = 0.70(130)

  0.25x +71.5 = 91 . . . simplify

  0.25x = 19.5 . . . . . . subtract 71.5

  x = 78 . . . . . . . . . . . divide by 0.25; amount of 80%

  130-78 = 52 . . . . amount of 55%

52 gallons of the 55% brand, and 78 gallons of the 80% brand must be used.

Air-USA has a policy of booking as many as 22 people on an airplane that can only seat 20 people. (Past studies have revealed that only 82% of the booked passengers actually show up for the flight.) a) Find the probability that if Air-USA books 22 people, not enough seats will be available. Round your answer to 4 decimal places. P ( X > 20 )

Answers

Answer:

The answer is "0.07404893".

Step-by-step explanation:

Applying the binomial distribution:

[tex]n = 22\\\\p= 82\%=0.82\\\\q = 1-0.82 = 0.18\\\\[/tex]

Calculating the probability for not enough seats:

[tex]=P(X>20)\\\\= P(21) + P(22)\\\\[/tex]

[tex]= \binom{22}{21} (0.82)^{21}(0.18)^1+ \binom{22}{22} (0.82)^{22}(0.18)[/tex]

[tex]=0 .06134598+ 0.01270295\\\\=0.07404893[/tex]

Question 19 of 28
Which of the following equations can be used to find the length of BC in the
triangle below?
B
10
А
30
с
A. BC = 30 + 10
B. (BC)2 = 102 + 302
C. BC = 30 - 10
D. (BC)2 = 302 - 102

Answers

Answer:

BC^2=10^2+30^2

Step-by-step explanation:

P=10B=30

Using pythagorean theorem

[tex]\\ \sf\longmapsto BC^2=10^2+30^2[/tex]

[tex]\\ \sf\longmapsto BC^2=100+300[/tex]

[tex]\\ \sf\longmapsto BC^2=400[/tex]

[tex]\\ \sf\longmapsto BC=\sqrt{400}[/tex]

[tex]\\ \sf\longmapsto BC=20[/tex]

14. A quadratic equation is graphed above.
Which of the following equations could be
paired with the graphed equation to create
a system of equations whose solution set is
comprised of the points (2,-2) and (-3, 3)?
A. y = x + 6
B. y = x - 6
C. y = X
D. y = -x

Answers

Answer:

D.

Step-by-step explanation:

2=-2,3=-3

2²=-2²,3²=3²

The length of a rectangle is 10 yd less than three times the width, and the area of the rectangle is 77 yd^2. Find the dimensions of the rectangle.

Answers

Answer:

W=7 and L=11

Step-by-step explanation:

We have two unknowns so we must create two equations.

First the problem states that  length of a rectangle is 10 yd less than three times the width so: L= 3w-10

Next we are given the area so: L X W = 77

Then solve for the variable algebraically. It is just a system of equations.

3W^2 - 10W - 77 = 0

(3W + 11)(W - 7) = 0

W = -11/3 and/or W=7

Discard the negative solution as the width of the rectangle cannot be less then 0.

So W=7

Plug that into the first equation.

3(7)-10= 11 so L=11

use induction method to prove that 1.2^2+2.3^2+3.4^2+...+r(r+1)^2= n(n+1)(3n^2+11n+10)/12

Answers

Base case (n = 1):

• left side = 1×2² = 4

• right side = 1×(1 + 1)×(3×1² + 11×1 + 10)/12 = 4

Induction hypothesis: Assume equality holds for n = k, so that

1×2² + 2×3² + 3×4² + … + k × (k + 1)² = k × (k + 1) × (3k ² + 11k + 10)/12

Induction step (n = k + 1):

1×2² + 2×3² + 3×4² + … + k × (k + 1)² + (k + 1) × (k + 2)²

= k × (k + 1) × (3k ² + 11k + 10)/12 + (k + 1) × (k + 2)²

= (k + 1)/12 × (k × (3k ² + 11k + 10) + 12 × (k + 2)²)

= (k + 1)/12 × ((3k ³ + 11k ² + 10k) + 12 × (k ² + 4k + 4))

= (k + 1)/12 × (3k ³ + 23k ² + 58k + 48)

= (k + 1)/12 × (3k ³ + 23k ² + 58k + 48)

On the right side, we want to end up with

(k + 1) × (k + 2) × (3 (k + 1) ² + 11 (k + 1) + 10)/12

which suggests that k + 2 should be factor of the cubic. Indeed, we have

3k ³ + 23k ² + 58k + 48 = (k + 2) (3k ² + 17k + 24)

and we can rewrite the remaining quadratic as

3k ² + 17k + 24 = 3 (k + 1)² + 11 (k + 1) + 10

so we would arrive at the desired conclusion.

To see how the above rewriting is possible, we want to find coefficients a, b, and c such that

3k ² + 17k + 24 = a (k + 1)² + b (k + 1) + c

Expand the right side and collect like powers of k :

3k ² + 17k + 24 = ak ² + (2a + b) k + a + b + c

==>   a = 3   and   2a + b = 17   and   a + b + c = 24

==>   a = 3, b = 11, c = 10

Find the area of a triangle with the given description. (Round your answer to one decimal place.)
a triangle with sides of length 14 and 28 and included angle 20°

Answers

9514 1404 393

Answer:

  67.0 square units

Step-by-step explanation:

The formula for the area is ...

  Area = 1/2ab·sin(C)

  Area = (1/2)(14)(28)sin(20°) ≈ 67.036 . . . . square units

The area of the triangle is about 67.0 square units.

Solve the system of equations.

6x−y=−14
2x−3y=6

whats the answer please C:

Answers

Answer:

Step-by-step explanation:

Please help on my hw

Answers

Answer:

b. The solution is a non empty set.

Step-by-step explanation:

There are no common elements.

g A gift shop sells 40 wind chimes per month at $110 each. The owners estimate that for each $11 increase in price, they will sell 2 fewer wind chimes per month. Find the price per wind chime that will maximize revenue.

Answers

Answer:

The price that maximizes the revenue is $165

Step-by-step explanation:

We can model the price as a function of sold units as a linear relationship.

Remember that a linear relationship is something like:

y = a*x + b

where a is the slope and b is the y-intercept.

We know that if the line passes through the points (x₁, y₁) and (x₂, y₂), then the slope can be written as:

a = (y₂ - y₁)/(x₂ - x₁)

For this line, we have the point (40, $110)

which means that to sell 40 units, the price must be $110

And we know that if the price increases by $11, then he will sell 2 units less.

Then we also have the point (38, $121)

So we know that our line passes through the points (40, $110) and  (38, $121)

Then the slope of the line is:

a = ($121 - $110)/(38 - 40) = $11/-2 = -$5.5

Then the equation of the line is:

p(x) = -$5.5*x + b

to find the value of b, we can use the point (40, $110)

This means that when x = 40, the price is $110

then:

p(40) = $110 = -$5.5*40 + b

            $110 = -$220 + b

       $110 + $220 = b

        $330 = b

Then the price equation is:

p(x) = -$5.5*x + $330

Now we want to find the maximum revenue.

The revenue for selling x items, each at the price p(x), is:

revenue = x*p(x)

replacing the p(x) by the equation we get:

revenue = x*(-$5.5*x + $330)

revenue = -$5.5*x^2 + $330*x

Now we want to find the x-value for the maximum revenue.

You can see that the revenue equation is a quadratic equation with a negative leading coefficient. This means that the maximum is at the vertex.

And remember that for a quadratic equation like:

y = a*x^2 + b*x + c

the x-value of the vertex is:

x = -b/2a

Then for our equation:

revenue = -$5.5*x^2 + $330*x

the x-vale of the vertex will be:

x = -$330/(2*-$5.5) = 30

x = 30

This means that the revenue is maximized when we sell 30 units.

And the price is p(x) evaluated in x = 30

p(30) = -$5.5*30 + $330 = $165

The price that maximizes the revenue is $165

If (4x-5) :(9x-5) = 3:8 find the value of x.​

Answers

Answer:

x is 5

Step-by-step explanation:

[tex] \frac{4x - 5}{9x - 5} = \frac{3}{8} \\ \\ 8(4x - 5) = 3(9x - 5) \\ 32x - 40 = 27x - 15 \\ 5x = 25 \\ x = \frac{25}{5} \\ \\ x = 5[/tex]

Step-by-step explanation:

as you can see as i solved above. all you need to do was to rationalize the both equations

On Halloween, a man presents a child with a bowl containing eight different pieces of candy. He tells her that she may have three pieces. How many choices does she have

Answers

Answer:

[tex]56[/tex] choices

Step-by-step explanation:

We know that we'll have to solve this problem with a permutation or a combination, but which one do we use? The answer is a combination because the order in which the child picks the candy does not matter.

To further demonstrate this, imagine I have 4 pieces of candy labeled A, B, C, and D. I could choose A, then C, then B or I could choose C, then B, then A, but in the end, I still have the same pieces, regardless of what order I pick them in. I hope that helps to understand why this problem will be solved with a combination.

Anyways, back to the solving! Remember that the combination formula is

[tex]_nC_r=\frac{n!}{r!(n-r)!}[/tex], where n is the number of objects in the sample (the number of objects you choose from) and r is the number of objects that are to be chosen.

In this case, [tex]n=8[/tex] and [tex]r=3[/tex]. Substituting these values into the formula gives us:

[tex]_8C_3=\frac{8!}{3!5!}[/tex]

[tex]= \frac{8*7*6*5*4*3*2*1}{3*2*1*5*4*3*2*1}[/tex] (Expand the factorials)

[tex]=\frac{8*7*6}{3*2*1}[/tex] (Cancel out [tex]5*4*3*2*1[/tex])

[tex]=\frac{8*7*6}{6}[/tex] (Evaluate denominator)

[tex]=8*7[/tex] (Cancel out [tex]6[/tex])

[tex]=56[/tex]

Therefore, the child has [tex]\bf56[/tex] different ways to pick the candies. Hope this helps!

Determine whether each relation is a function. Give the domain and range for each relation.
{(3, 4), (3, 5), (4, 4), (4, 5)}

Answers

Answer:

Not a function

Domain: {3,4}

Range: {4,5}

Step-by-step explanation:

A function is a relation where each input has its own output. In other words if the x value has multiple corresponding y values then the relation is not a function

For the relation given {(3, 4), (3, 5), (4, 4), (4, 5)} the x value 3 and 4 have more than one corresponding y value therefore the relation shown is not a function

Now let's find the domain and range.

Domain is the set of x values in a relation.

The x values of the given relation are 3 and 4 so the domain is {3,4}

The range is the set of y values in a relation

The y value of the given relation include 4 and 5

So the range would be {4,5}

Notes:

The values of x and y should be written from least to greatest when writing them out as domain and range.

They should be written inside of brackets

Do not repeat numbers when writing the domain and range

Other Questions
Instructions: State what additional information is required in orderto know that the triangles in the image below are congruent for thereason givenReasory. SAS Postulate Hi!! need help pls xx What are the significance and effects of revolutions in a country's history? Look at the sentence that follows. When translated to Spanish, would it use the subjunctive?It's certain that Marcos is coming to the party.No or yes. What is the order of rotational symmetry for the figure?A. 1B. 2C. 3D. 4 or more How many points appear in this figure? A spinner numbered 1 to 6 is spun ten times, giving the results : 2, 5, 3, 5, 1, 2, 3, 3, 4, 6. The experimental probability of getting a 2 is: Write sentences in simple past. DO NOT USE CONTRACTIONS There is one example: EXAMPLE: 0. I / dinner / cook ANSWER: I cooked dinner1. Jim / his head / cover2. She / the chapter / copy3. not / The clouds / disappear4. He / to our question / refer5. not / we / our punishment / escape A job order costing system does which of the following? Select one: A. Is used to determine period costs in a service company B. Is used to determine unit costs when products are manufactured in a continuous flow process C. Allocates manufacturing costs to individual jobs to determine unit costs D. Both A and B E. None of the above Which area did the Bantu not live in?A. Great Lakes regionB. Great Rift ValleyC. North African savannahD. Equatorial rain forest Find the slope (m) of the line using the origin, (0,0) and the point (2,8) Describe a very morning on way to school find the square root and of 1+i3 I need help ASAP!!! Please explain your answer 14 over 17 as a decimal rounded to the nearest tenth A scientist started with a sample of 8 cells. The sample increased as shown in the table.Assume that the pattern in the table continues. Which equation can be solved for t, the time in hours when the number of cells will reach 100,000?A 8t4=100,000B 84t=100,000C 4t8=100,000D 48t=100,000 What was an important effect of the Haitian Revolution? Marvin Industries owns a piece of equipment with a cost of $78,000 and accumulated depreciation of $51,000. The equipment is sold for $30,000 cash. The amount that should be reported as a cash inflow from investing activities is: Which of the following statements is generally true about "resilient" people? a) They get confused easily. b) are open to trying new things. c) They prefer to work with people who are similar to themselves. d) They prefer to focus on many tasks simultaneously. A rhombus has an area of 5 square meters and a side length of 3 meters. In another similar rhombus, the length of a side is 9 meters. What is the area of the second rhombus?(A) 30 square meters(B) 45 square meters(C) 60 square meters(D) 75 square meters